1956 AHSME Problems/Problem 14

Problem 14

The points $A,B,C$ are on a circle $O$. The tangent line at $A$ and the secant $BC$ intersect at $P, B$ lying between $C$ and $P$. If $\overline{BC} = 20$ and $\overline{PA} = 10\sqrt {3}$, then $\overline{PB}$ equals:

$\textbf{(A)}\ 5 \qquad\textbf{(B)}\ 10 \qquad\textbf{(C)}\ 10\sqrt {3} \qquad\textbf{(D)}\ 20 \qquad\textbf{(E)}\ 30$

Solution

Because $PA$ is a tangent line, angle $\angle OAP$ is a right angle. Drop a perpendicular from $O$ to $BC$ at $E.$ We find that $BE = EC = 10.$

Let $AR = r$ and $PE = a$. We now have a system of equations.

\[(10\sqrt{3})^2+r^2=PR^2\] \[(\sqrt{r^2-10^2})^2+PE^2=r^2-10^2+PE^2=PR^2\]

Set them equal to each other and solve.

\[(10\sqrt{3})^2+r^2=r^2-10^2+PE^2\] \[(10\sqrt{3})^2=-10^2+PE^2\] \[400=PE^2\] \[20=PE\]

We know $BE = 10$, so $PE = 20 - 10 = 10$, which is $\boxed{B}$.

~Revised by MC413551

See Also

1956 AHSC (ProblemsAnswer KeyResources)
Preceded by
Problem 13
Followed by
Problem 15
1 2 3 4 5 6 7 8 9 10 11 12 13 14 15 16 17 18 19 20 21 22 23 24 25 26 27 28 29 30 31 32 33 34 35 36 37 38 39 40 41 42 43 44 45 46 47 48 49 50
All AHSME Problems and Solutions

The problems on this page are copyrighted by the Mathematical Association of America's American Mathematics Competitions. AMC logo.png